Đến nội dung

Saturina

Saturina

Đăng ký: 24-10-2022
Offline Đăng nhập: 24-02-2024 - 16:30
***--

#743607 Chứng minh ba điểm E, F, H thẳng hàng.

Gửi bởi Saturina trong 16-02-2024 - 09:43

Cho tam giác ABC nhọn có trực tâm H , nội tiếp đường tròn (O).Đường
thẳng CH cắt AB tại D . Đường thẳng qua D vuông góc với OD , cắt đường thẳng
BC tại E. Đường tròn ngoại tiếp tam giác BCH cắt đường thẳng AB tại F ( F không
trùng B ). Chứng minh ba điểm E, F, H thẳng hàng.



#743597 Hỏi có thể xây dựng mà mỗi phòng có đúng hai cửa hay không?

Gửi bởi Saturina trong 16-02-2024 - 09:16

Một công ty muốn xây một công trình có kích thước 2021 x 2023 gồm
4088483 phòng, mỗi phòng có kích thước
1 x 1, một số phòng kề nhau (chung cạnh) được nối với
nhau bằng một cửa giữa hai phòng. Hỏi có thể xây dựng mà mỗi phòng có đúng hai cửa hay
không?

 




#743596 Có 8 học sinh tham gia làm một bài kiểm tra trắc nghiệm. Sau khi kiểm tra, th...

Gửi bởi Saturina trong 16-02-2024 - 09:15

Có 8 học sinh tham gia làm một bài kiểm tra trắc nghiệm. Đề thi
chung cho tất cả học sinh, có
n câu hỏi, mỗi câu hỏi có 4 phương án trả lời, học
sinh chỉ được chọn một phương án cho mỗi câu hỏi. Sau khi kiểm tra, thấy rằng hai
học sinh bất kì có chung nhiều nhất một câu trả lời.
a) Với
n = 4, hãy chỉ ra một ví dụ về phương án trả lời các câu hỏi của 8 học sinh.
b) Tìm giá trị lớn nhất của số câu hỏi trong bài kiểm tra.

 




#741039 Công thức tính tổng $\sum_{i=0}^{m} \binom...

Gửi bởi Saturina trong 14-08-2023 - 21:45

Mình đoán ý bạn ở câu hỏi trên là tổng đó có một công thức dạng đóng nào không, theo kiểu $\sum^{n}_{k=1} k = \frac{n(n+1)}{2}$. Câu trả lời là: đến nay vẫn chưa có một công thức dạng đóng nào cho tổng $\displaystyle\sum^{m}_{i=0}\binom{n}{i}$.

 

Mình dám nói là chưa có công thức dạng đóng nào vì mình đã tìm kiếm hồi lâu với Google, theo câu tìm kiếm: "partial sum of binomial coefficients". Mặt khác, tổng trên có ý nghĩa thế này: tổng các chuỗi nhị phân độ dài $n$, trong đó số $1$ xuất hiện không quá $m$ lần. Với tổng trên, người ta quan tâm đến cận trên của nó, chưa tim ra (và rất chắc là không có) công thức dạng đóng. "Chấp niệm" này nên bỏ qua thôi. Trong thực tế, có nhiều thứ không có công thức dạng đóng.

 

Còn để tính nhanh, cũng chưa có một cách tính trên giấy mà hiệu quả, ngoại trừ rất ít trường hợp đặc biệt của $m$. Cách tính nhanh nhất hiện giờ là viết một chương trình nhỏ (một hàm) để tính tổng trên. Nếu bạn làm theo cách này thì hãy cẩn thận với giai thừa vì giai thừa tăng rất nhanh.

 

Mình nghĩ là bạn có một mục tiêu hoặc vấn đề toán học/khoa học nào khác để mà dẫn tới nhu cầu tính tổng trên một cách hiệu quả. Nếu là vậy, việc thảo luận vấn đề gốc có thể sẽ có ích hơn, bởi biết đâu lại có một hướng đi không cụt.

mình cảm ơn bạn nhé, thực ra mình đang có một vấn đề về lập trình, nhưng vấn đề là việc tính tổng đó mất nhiều thời gian ( thời gian của máy tính ấy ) nên là mình muốn xem có công thức nào không :)))




#741033 Công thức tính tổng $\sum_{i=0}^{m} \binom...

Gửi bởi Saturina trong 14-08-2023 - 09:24

Cho em hỏi có công thức nào tính nhanh tổng này không ạ

$$\sum_{i=0}^{m} \binom{n}{i}$$  với ($m \le n$)




#737833 Đáp án VMO 2023

Gửi bởi Saturina trong 18-03-2023 - 14:52

Bộ GD-ĐT đã chinh thức công bố đáp án đề thi VMO 2023.

Xem tại: https://drive.google...SU0rlasFpaxtbwE

P/s: Đáp án Bộ làm mà sao em xem nó mờ thế nhỉ




#737831 Thông báo về việc bảo trì diễn đàn

Gửi bởi Saturina trong 17-03-2023 - 23:31

à mà anh @Nesbit ơi, em nghĩ nên tăng cái giới hạn tin nhắn private message lên tầm khoảng 100 hay 1000 gì đấy ạ, chứ em thấy 30 tin nhắn hơi ít =)




#737694 [HOT HOT HOT] ĐÃ CÓ KẾT QUẢ THI HSG QUỐC GIA 2023

Gửi bởi Saturina trong 13-03-2023 - 20:18

[HOT HÒN HỌT] ĐÃ CÓ KẾT QUẢ THI HSG QUỐC GIA 2023
Xem tại đây: 
https://drive.google...UenqZ8CyidSSNxc




#737693 Thông báo về việc bảo trì diễn đàn

Gửi bởi Saturina trong 13-03-2023 - 20:11

Cảm ơn @Saturina. Thực ra thì anh cũng muốn làm từ lâu rồi nhưng bận quá. Vừa rồi phải chuyển host nên anh nhân tiện làm luôn vì đằng nào cũng phải mất thời gian.

Em thấy giao diện mới thế nào? Có chỗ nào chưa đẹp thì cứ báo hết vào đây cho anh nhé, anh sẽ chỉnh dần.

Em nghĩ anh nên đưa ra trang chủ thêm một chỗ để xem trạng thái các thành viên ý ạ, chứ em vẫn chưa biết tìm ở đâu mà phải gõ link:
https://diendantoanh...g/statuses/all/
Với lại em nghĩ những box nào mà nhiều người thảo luận và cần thiết cho mọi người như là Đại số, Hình học, Số học, Tổ hợp, Đại số tuyến tính, Bất đẳng thức thì em nghĩ cũng nên đưa ra ngoài trang chủ để mọi người click vô phát là đến luôn ạ
Hoặc anh làm rồi nhưng mà em không biết  :D  :D




#737633 Thông báo về việc bảo trì diễn đàn

Gửi bởi Saturina trong 10-03-2023 - 22:23

Các bạn thân mến,

 

Đúng ra ngày hôm qua thì việc bảo trì diễn đàn đã hoàn thành. Tuy nhiên do một số trục trặc kĩ thuật nên diễn đàn đã hoạt động không ổn định lắm (bao gồm cả chức năng backup do vừa đổi server), và điều này dẫn tới việc những nội dung được đăng và thành viên đăng ký ngày 8 và 9/3 đã không được lưu lại. Xin thay mặt BQT gửi lời xin lỗi tới tất cả các bạn, đặc biệt là những thành viên mới đăng ký và những bạn đã đăng bài (Nesbit không nhớ hết nhưng có thể kể ra vài thành viên như @chanhquocnghiem, @Nobodyv3, @nmlinh16, @Ruka, @Nesbitttt...).

 

Để bù lại Nesbit đã gắng bổ sung thêm chức năng "nhắc đến" cho diễn đàn: các bạn có thể dùng dấu @ rồi gõ tên đăng nhập của một thành viên để đề cập đến thành viên đó trong bài viết hoặc trong cập nhật trạng thái của mình. Một hạn chế đó là lúc gõ @ thì hệ thống chưa gợi ý được tên thành viên cho bạn mà bạn phải tự gõ hết tên (chức năng gợi ý này sẽ có trong ít tháng nữa lúc diễn đàn được nâng cấp).

 

Tổng kết lại thì lần bảo trì này mang đến những thay đổi sau đây:

 

1. Giao diện mới hiện đại hơn.

2. Bộ máy xử lí công thức Toán được nâng cấp, nhanh hơn nhiều so với trước đây.

3. Chức năng đề cập đến thành viên khác trong bài viết hoặc cập nhật trạng thái.

 

Nếu các bạn có góp ý hoặc thấy có lỗi nào thì xin vui lòng báo cho BQT trong chủ đề này.

 

Cảm ơn các bạn đã gắng bó với Diễn đàn Toán học.

 

Thay mặt BQT

Em cảm ơn anh đã chú ý đến lời góp ý của em ạ

Em không ngờ anh lại thực hiện nhanh thế :icon6:




#737590 đa thức bất khả quy

Gửi bởi Saturina trong 06-03-2023 - 22:56

ĐA THỨC BẤT KHẢ QUY

(Thầy Vương Trung Dũng  giáo viên trường PTNK TP Hồ Chí Minh)

1. Giới thiệu sơ lược 

Đa thức bất khả qui là một vấn đề kinh điển trong đa thức nói riêng và trong toán học nói chung. Các bài toán về đa thức bất khả qui cũng thường xuyên xuất hiện trong các kì thi Olympic về toán. Người ta quan tâm nhiều nhất về tính bất khả qui của một đa thức trên vành $\mathbb{Z}[x]$ và $\mathbb{Q}[x]$. Có nhiều cách để kiểm tra tính bất khả qui của một đa thức loại này chẳng hạn như dùng trực tiếp định nghĩa hoặc dùng các tiêu chuẩn như tiêu chuẩn Eisenstein, tiêu chuẩn Perron, tiêu chuẩn Cohn, tiêu chuẩn Dumas… tuy nhiên bài viết này chỉ đề cập đến hai phương pháp thường được sử dụng nhất là sử dụng trực tiếp định nghĩa và tiêu chuẩn Eisenstein và các dạng mở rộng của nó cùng với đó là một kĩ thuật tối quan trọng là rút gọn theo một modulo nguyên tố $p$. Các tiêu chuẩn khác hi vọng sẽ có dịp trình bày trong một bài viết khác.

Trong tài liệu này ta qui ước $\mathbb{Z}_p=\mathbb{Z}/p\mathbb{Z}$ và $\mathbb{K}$ là một trong các tập $\mathbb{Z},\ \mathbb{Q},\ \mathbb{R}, \ \mathbb{Z}_p$. Khi đó, $ \mathbb{K}[x]$ (tương ứng $ \mathbb{K}[x,y]$) là các vành đa thức một biến (tương ứng 2 biến) có hệ số trong $ \mathbb{K}$.

Định nghĩa 1.1:  Đa thức $P(x)$ trong vành $\mathbb{K}[x]$ được gọi là khả qui trên $\mathbb{K}$ nếu $P(x)=f(x).g(x)$ trong đó $f(x), g(x)$ là các đa thức không khả nghịch trong $\mathbb{K}[x]$. Đa thức $P(x)$ được gọi là bất khả qui nếu $P(x)$ không khả nghịch và không khả qui.

Nói riêng, khi $\mathbb{K}$ là một trường thì một đa thức $P(x) \in \mathbb{K}[x]$ có bậc dương được gọi là khả qui trên $\mathbb{K}$ nếu có thể phân tích được thành tích hai đa thức có bậc dương trong $\mathbb{K}[x]$, ngược lại $P(x)$ được gọi là bất khả qui trên $\mathbb{K}$.

Định lí Gauss 1.1: Các vành đa thức

  • $\mathbb{R}[x], \ \mathbb{C}[x],\ \mathbb{Q}[x], \ \mathbb{Z}_p[x]$
  •  $\mathbb{Z}[x], \ \mathbb{Z}[x,y], \ \mathbb{Q}[x,y]…$

là có sự phân tích duy nhất thành các nhân tử bất khả qui và sự phân tích này là duy nhất. Nói riêng các khái niệm về đa thức bất khả qui, ước chung lớn nhất, bội chung nhỏ nhất vẫn còn đúng trên các vành này.

Lưu ý: Trong trường hợp $1$ ở trên là các đa thức có hệ số trên trường nên trên đó thuật toán Euclid hay định lí Bezout vẫn còn đúng nhưng trường hợp $2$ thì không.

2. Tính bất khả qui trên $\mathbb{C}[x]$ và $\mathbb{R}[x]$

Định lí 2.1: Mọi đa thức có bậc lớn hơn 1 đều khả qui trên $\mathbb{C}[x]$.

Chứng minh

Giả sử $degP>1$. Ta thừa nhận định lí cơ bản của đại số “Mọi đa thức $P(x) \in \mathbb{C}[x]$ có bậc lớn hơn 1 đều có ít nhất một nghiệm trên $\mathbb{C}$”. Khi đó $P(x)$ có nghiệm $x_0 \in \mathbb{C}$ nên theo Định lí Bezout $$P(x)=(x-x_0)Q(x),$$

trong đó $deg\ge 1$ nên $P(x)$ khả qui trên $\mathbb{C}[x].$

Định lí 2.2: Mọi đa thức có hệ số thực bậc lớn hơn 2 đều khả qui trên $\mathbb{R}[x]$. Nói riêng một đa thức là bất khả qui trên $\mathbb{R}[x]$ khi và chỉ khi nó là đa thức bậc nhất hoặc bậc 2 vô nghiệm.

Chứng minh

Giả sử $P \in \mathbb{R}[x]$ và $deg P >2$.

  • Nếu $\deg P$ lẻ thì $P$ có ít nhất một nghiệm thực nên nó khả qui.
  • Nếu $\deg P$ chẵn thì $P$ có một nghiệm phức $\alpha$, khi đó $\overline{\alpha}$ cũng là nghiệm của $P$ và do đó $P(x)=(x-\alpha)(x-\overline{\alpha})Q(x)$ là khả qui.

3. Tính bất khả qui trên $\mathbb{Z}[x]$ và $ \mathbb{Q}[x]$

Qua Định lí 2.1 và Định lí 2.2 ta thấy nếu $\mathbb{K}=\mathbb{C},\ \mathbb{R}$ thì tính bất khả quy là đơn giản nên ta quan tâm đến trường hợp $\mathbb{K}=\mathbb{Z}, \ \mathbb{Q}.$ Thật may mắn là bổ đề Gauss mà ta trình bày sau đây sẽ cho ta một sự tương ứng về tính bất khả qui của một đa thức hệ số nguyên trên $\mathbb{Z}[x]$ và $\mathbb{Q}[x]$.

Định nghĩa 3.1: Cho đa thức $P(x)=a_nx^n+a_{n-1}x^{n-1}+…+a_1x+a_0 \in \mathbb{Z}[x]$, đa thức $P$ được gọi là nguyên bản nếu $gcd(a_n,…,a_0)=1$

Mệnh đề 3.1: Tích của hai đa thức nguyên bản là một đa thức nguyên bản.

Mệnh đề 3.2: Mọi đa thức $P \in \mathbb{Q}[x]$ đều viết được dưới dạng $P=cP_0(x)$, trong đó $P_0$ là một đa thức nguyên bản và $c_0 \in \mathbb{Q}.$

Định lí 3.1 (Bổ đề Gauss):  Một đa thức hệ số nguyên, có bậc dương bất khả qui trong $\mathbb{Q}[x]$ khi và chỉ khi nó bất khả qui trong $\mathbb{Z}[x]$.

Chứng minh

Hiển nhiên nếu $P(x)$ bất khả qui trên $\mathbb{Q}[x]$ sẽ bất khả qui trên $\mathbb{Z}[x]$.

Ngược lại giả sử $P(x)$ bất khả qui trên $\mathbb{Z}[x]$ mà

$P(x)=P_1(x)P_2(x)$, với $P_1, P_2 \in \mathbb{Q}[x]$ và $1\le deg \ P_1, degP_2 \le deg\ P$.

Khi đó ta viết lại $P_1=\dfrac{a_1}{b_1}Q_1(x), P_2=\dfrac{a_2}{b_2}Q_2(x)$, với $(a_i,b_i)=1$ và $Q_i$ nguyên bản, $i \in \{1,2\}$.

Suy ra $P(x)=\dfrac{a_1a_2}{b_1b_2}Q_1(x)Q_2(x)=\dfrac{p}{q}Q_1(x)Q_2(x),$ trong đó $p=a_1b_1, q=a_2b_2$ và $ (p,q)=1.$

Do $P\in \mathbb{Z}[x]$ nên các hệ số của $Q_1(x)Q_2(x)$ phải chia hết cho $q$ điều này trái với tính nguyên bản của $Q_1(x)Q_2(x)$.

Từ đó ta có điều phải chứng minh.

Định lí 3.2: Cho $P(x)=a_nx^n+a_{n-1}x^{n-1}+…+a_1x+a_0 \in \mathbb{Z}[x]$. Giả sử $P$ có nghiệm hữu tỉ $x=\dfrac{p}{q}$ với $(p,q)=1$. Khi đó, $p$ là ước của $a_0$ còn $q$ là ước của $a_n.$ Nói riêng, mọi nghiệm hữu tỉ của một đa thức monic (đơn khởi, hệ số của bậc cao nhất bằng $\pm1$) với hệ số nguyên đều là nghiệm nguyên.

Chứng minh

Giả sử $P(x)$ có nghiệm hữu tỉ $\dfrac{p}{q},$ với $ (p,q)=1$. Khi đó $$a_n(\dfrac{p}{q})^n+…+a_1. \dfrac{p}{q}+a_0=0,$$

qui đồng mẫu số ta được $$a_np^n+…+a_0q^n=0.$$

Vì vế phải chia hết cho $p$ nên vế trái chia hết cho $p$, từ đó suy ra $a_0q^n$ chia hết cho $p$, lại có $(q^n,p)=1$ nên $a_0$ chia hết cho $p$. Lập luận tương tự ta được $a_n$ chia hết cho $q$.

Đinh lí 3.3: Cho $P \in \mathbb{Q}[x]$ có bậc 2 hoặc 3. Khi đó, $P(x)$ là bất khả qui khi và khi khi $P(x)$ không có nghiệm hữu tỉ.

Chứng minh

Hiển nhiên nếu $P$ có nghiệm hữu tỉ thì nó khả qui.

Đảo lại, nếu $P$ khả qui thì $P$ phân tích được thành tích của hai đa thức hữu tỉ.

Điều kiện bậc của $P$ bằng 2 hoặc 3 chứng tỏ một trong hai nhân tử trên phải có bậc 1.

Từ đó suy ra $f$ có nghiệm hữu tỉ.

Lưu ý: Định lí trên vẫn còn đúng nếu ta thay $\mathbb{Q}$ bởi một trường $\mathbb{K}$ bất kì. Tức là, đa thức $f \in \mathbb{K}[x]$ với bậc bằng 2 hoặc 3 là bất khả qui nếu và chỉ nếu nó không có nghiệm trong $\mathbb{K}.$

Dưới đây là một số ví dụ

Ví dụ 3.1 (Định lí Schur): Cho các số nguyên phân biệt $a_1, a_2,…,a_n$. Khi đó đa thức

a) $f(x)=(x-a_1)(x-a_2)…(x-a_n)-1$ là bất khả qui trên $\mathbb{Q}[x]$.

b) $f(x)=(x-a_1)(x-a_2)…(x-a_n)+1$ là bất khả qui trên $\mathbb{Q}[x]$ ngoại trừ các trường hợp

  • $(x-a)(x-a-2)+1=(x-a-1)^2$,
  • $(x-a)(x-a-1)(x-a-2)(x-a-3)+1=[(x-a-1)(x-a-2)+1]^2.$

Giải

a) Giả sử $(x-a_1)(x-a_2)…(x-a_n)-1=g(x)h(x)$, với $1 \le deg \ f, \deg \ g \le n-1$ và $g, h \in \mathbb{Z}[x].$

Ta có $g(a_i)h(a_i)=-1$ với mọi $i$, từ đó do $g(a_i), h(a_i)$ là các số nguyên nên $ (g+h)(a_i)=0,$ với mọi $i=1,2,…,n.$

Nhưng vì $deg \ (g+h) \le n-1$ triệt tiêu tại $n$ giá trị phân biệt nên $g \equiv – h$.

Từ đó $$(x-a_1)(x-a_2)…(x-a_n)-1=-(g(x))^2.$$

Đẳng thức trên là vô lí vì hệ số cao nhất ở hai vế trái dấu.

b) Lập luận hoàn toàn như trên, giả sử $f(x)$ là khả qui, bằng một phép đổi biến đơn giản ta hoàn toàn có thể viết lại $f$ dưới dạng $$f(x)=x(x-a_1)(x-a_2)…(x-a_{n-1})+1=g(x).h(x),$$ trong đó $0<a_1<a_2<…<a_{n-1}$ và $1 \le g, h \in \mathbb{Z}, \deg(g), \deg(h) \le n-1$.

Từ đẳng thức $g(a_i)h(a_i)=1$ ta suy ra $g(a_i)=h(a_i)= \pm 1$ với mọi $i$ và đẳng thức này xảy ra tại $n$ giá trị phân biệt. Điều đó dẫn dến $g(x)=h(x)$ và ta có $f(x)=g^2(x)$.

Nói cách khác, $\deg(f)=n$ là một số chẵn. Khi đó $f(\dfrac{1}{2})=\dfrac{1}{2}(\dfrac{1}{2}-a_1)…(\dfrac{1}{2}-a_{n-1})+1=1-\dfrac{1}{2^n}(2a_1-1) \ldots (2a_{n-1}-1) \le 1-\dfrac{1}{2^n}1.3 \ldots (2n-3) <0$ với mọi $n \ge 6$ (vô lí). Như vậy $n=2$ hoặc $n=4$.

  • Nếu $n=2$ thì $f(\dfrac{1}{2})=1-\dfrac{1}{4}(2a_1-1) \Rightarrow a_1 \le \dfrac{5}{2}$ và do đó $a_1=1, 2$. Giá trị $a_1$ cho ta $f(x)=x(x-1)+1$ là bất khả qui và $a_1=2$ cho ta $f(x)=x(x-2)+1=(x-1)^2$ là khả qui.
  • Nếu $n=4$ thì $f(\dfrac{1}{2})=1-\dfrac{1}{16}(2a_1-1)(2a_2-1)(2a_3-1) \Rightarrow 0 \le \dfrac{1}{16}(2-1)(3-1)(2a_3-1) \Rightarrow a_3 \le \dfrac{19}{6}$. Xét trường hợp $a_1=1, a_2=2, a_3=3$ ta được $$f(x)=x(x-1)(x-2)(x-3)+1=(x^2-3x+1)^2$$ là khả qui.

Bài toán được chứng minh xong.

Ví dụ 3.2: Cho $a_1, a_2,…,a_n$ là các số nguyên dương phân biệt. Chứng minh rằng đa thức $$P(x)=(x-a_1)^2(x-a_2)^2…(x-a_n)^2+1$$ là bất khả qui trên $\mathbb{Z}.$

Giải

Giả sử $P(x)$ là khả qui, tức tồn tại hai đa thức $G(x), H(x) \in \mathbb{Z}[x]$ có bậc không bé hơn 1 sao cho $P(x)=G(x).H(x)$.

Ta có $P(a_i)=G(a_i).H(a_i)$ với $i=1,2,…,n$ nên $G(a_i)=H(a_i)= \pm 1.$ Ta xét các trường hợp

  • Nếu $deg \ G= \ deg \ H$ thì $deg(G-H) \le n-1 \Rightarrow G \equiv H.$ Từ đó $P(x)=(G(x))^2 \Leftrightarrow 1= \Big(G(x)-(x-a_1)…(x-a_n) \Big)\Big(G(x)+(x-a_1)…(x-a_n) \Big)$ , vô lí vì bậc vế phải luôn không nhỏ thua 1.
  • Nếu $degH<deg G$ thì $degH<n$ mà $H(a_i)= \pm1, i=1,2,…,n$ dẫn đến $H$ là đa thức hằng, vô lí.

Vậy $P(x)$ bất khả qui.

4. Rút gọn modulo $p$ nguyên tố

Kĩ thuật rút gọn modulo $p$ nguyên tố là một kĩ thuật tối quan trọng trong việc chứng minh một đa thức là bất khả qui trên $\mathbb{Z}$. Nó đưa các hệ số từ một trường vô hạn các phần tử về một trường hữu hạn các phần từ, từ đó các tính toán của ta có thể được đơn giản hơn.

Định nghĩa 4.1: Cho $P(x)= \sum \limits_{i=0}^n a_ix_i \in \mathbb{Z}[x], a_n \ne 0$ và $p$ là số nguyên tố. Giả sử $p$ không phải là ước của $gcd(a_1,a_2,…,a_n)$. Ta kí hiệu $\overline{P}$ là đa thức nhận được từ $P$ bằng cách rút gọn các hệ số theo modulo $p$ (lúc này $P(x) \in \mathbb{Z}_p[x]$). Khi đó ta gọi $\overline{P}$ là \textit{đa thức rút gọn theo modulo} $p$ của $P.$

Từ định nghĩa trên ta có sự kiện sau là hiển nhiên $$\overline{P+Q}=\overline{P}+\overline{Q}$$

$$\overline{PQ}=\overline{P}. \ \overline{Q}$$

Định nghĩa 4.2: Nếu đa thức rút gọn modulo $p$ của P bất khả qui thì ta nói đa thức $P$ bất khả qui $mod \ p.$

Định lí 4.1: Với mỗi $P(x) \in \mathbb{Z}[x]$, tồn tại các đa thức $P_1(x), P_2(x), …, P_k(x) \in \mathbb{Z}_p[x]$ sao cho $$\overline{P}(x)=P_1(x).P_2(x)…P_k(x),$$

sự phân tích này là duy nhất theo modulo $p$.

Định lí 4.2: Cho $P(x)= \sum \limits_{i=0}^n a_ix_i \in \mathbb{Z}[x], a_n \ne 0$ và $p$ không là ước của $a_n$. Khi đó, nếu $P(x)$ là bất khả qui $mod \ p$ thì $P(x)$ là bất khả qui. Điều ngược lại của định lí nói chung không đúng.

Chứng minh

Giả sử $P(x)=a_nx^n+a_{n-1}x^{n-1}+…+a_1x+a_0 \in \mathbb{Z}[x]$ và $p$ không là ước của $a_n$.

Giả sử $P(x)=f(x).g(x)$ với $f, g \in \mathbb{Z}[x]$ với $deg \ f, g \ge 1$.

Khi đó $\overline{P}=\overline{f}. \overline{g}$. Do $p$ không là ước của $a_n$ nên bậc của các đa thức $P, f, g$ không thay đổi sau khi rút gọn theo modulo $p$.

Điều này chứng tỏ $\overline{P}$ khả qui theo modulo $p$, vô lí. Ta có điều phải chứng minh.

Ngược lại dễ thấy đa thức $P(x)=x^4+1$ bất khả qui trên $\mathbb{Z}[x]$ nhưng khả qui modulo $p$ với mọi số nguyên tố $p$.

Ví dụ 4.1: Chứng minh đa thức $P(x)=x^5+4x^4+2x^3+5x^2-7$ là bất khả qui.

Giải

  •  Rút gọn theo modulo 2 ta được $\overline{P}(x)=x^5+x^2+1.$
  •  Giả sử $\overline{P}(x)=f(x). g(x)$, với $f, g \in \mathbb{Z}_2$.
  •  Nếu $deg \ f=1$ hoặc $deg \ g=1$ dễ thấy là vô lí vì $\overline{P}$ không có nghiệm trong $\mathbb{Z}_2$.
  •  Suy ra $\overline{P}(x)=(x^2+ax+b)(x^3+cx^2+dx+e)$, với $a,b,c,d,e \in \mathbb{Z}_2$. Đồng nhất hệ số hai vế ta được điều vô lí. Từ đó suy ra điều phải chứng minh.

Ta có thể liệt kê ra các đa thức bất khả qui modulo 2 trong một số trường hợp bậc nhỏ như sau

  • Trường hợp $n=1$ gồm các đa thức: $x, x+1$.
  •  Trường hợp $n=2$ chỉ gồm một đa thức: $1+x+x^2$.
  •  Trường hợp $n=3$ gồm các đa thức: $1+x+x^3, 1+x^2+x^3$.
  •  Trường hợp $n=4$ gồm các đa thức: $1+x+x^4, 1+x+x^2+x^3+x^4$.
  •  Trường hợp $n=5$ gồm các đa thức:

$1+x+x^2+x^4+x^5$,

$1+x+x^3+x^4+x^5$,

$1+x^2+x^3+x^4+x^5$,

$1+x+x^2+x^3+x^5$,

$1+x^3+x^5, 1+x^2+x^5.$

5. Tiêu chuẩn Eisenstein và một số dạng mở rộng

Khi kiểm tra đa tính bất khả qui của một đa thức trên $\mathbb{Z}[x]$ tiêu chuẩn Eisenstein cung cấp cho ta một công cụ hiệu quả.

Định lí 5.1 (Tiêu chuẩn Eisenstein): Cho đa thức $P(x)= \sum \limits_{i=0}^na_ix^i \in \mathbb{Z}[x], a_n \ne 0$. Khi đó nếu tồn tại số nguyên tố $p$ thỏa đồng thời các điều kiện

  • $p$ không là ước của $a_n$;
  • $p$ là ước của $a_i$ với mọi $i\in \{1,2,…,n-1\}$;
  • $p^2$ không là ước của $a_0.$

Khi đó $P(x)$ là đa thức bất khả qui trên $\mathbb{Q}[x].$

Chứng minh

Có rất nhiều cách chứng minh cho tiêu chuẩn Eisenstein, ở đây ta sẽ trình bày chứng minh bằng cách rút gọn theo một modulo $p$ nguyên tố bất kì.

Giả sử $f$ khả qui, tức $f(x)=g(x).h(x)$, với $f, g \in \mathbb{Z}[x]$ và $deg \ f, g \ge 1.$ Rút gọn theo modulo $p$ nguyên tố ta được đẳng thức trong $\mathbb{Z}_p[x]$ dưới dạng $$\overline{f}=\overline{g}. \overline{h}.$$

Từ điều kiện $p$ là ước của $a_0, …, a_{n-1}$ nhưng không là ước của $a_n$ ta suy ra $\overline{f}=\overline{a_n}x^n.$

Từ đó suy ra $\overline{g}= \overline{b_k}x^k, \overline{h}=\overline{b_m}x^m$. Điều này có nghĩa là $\overline{b_0} \equiv…\equiv \overline{b_{k-1}} \equiv \overline{c_0} \equiv… \equiv \overline{c_{m-1}} \equiv 0 \ mod(p)$. Nhưng khi đó $a_0=b_0c_0 \equiv0 \ (mod \ p^2)$ (vô lí). Từ đó ta có điều phải chứng minh. $\square$

Ví dụ 5.1: Chứng minh đa thức $P(x)=x^4-x^2+2x+1$ bất khả qui trên $\mathbb{Z}$.

Giải

Đặt $Q(x)=P(x+1)=x^4+3x^3+3x^2+3x+3$. Khi đó theo tiêu chuẩn Eisenstein với $p=3$ ta có điều phải chứng minh.

Định lí 5.2 (Dạng mở rộng thứ nhất của tiêu chuẩn Eisenstein):

Cho $f(x)=a_nx^n+a_{n-1}x^{n-1}+…+a_1x+a_0 \in \mathbb{Z}[x]$. Giả sử tồn tại số nguyên tố $p$ thỏa mãn với một số tự nhiên $k \le n$ nào đó mà

  • p không là ước của $a_k$;
  • $p$ là ước của $a_0, …, a_{k-1}$;
  • $p^2$ không là ước của $a_0$.

Thế thì $f(x)$ có một nhân tử bất khả qui bậc $ \ge k$ ( và do đó nếu không bất khả qui sẽ có một nhân tử bậc $\le n-k$)

Chứng minh: Bạn đọc có thể tự chứng minh như trong trường hợp nguyên bản của định lí.

Ví dụ 5.2: Chứng minh đa thức $f(x)=x^{101}+101x^{100}+102$ là bất khả qui.

Giải

Áp dụng tiêu chuẩn Eisenstein mở rộng cho $p=2, k=100$ ta thấy, nếu $f$ là khả qui thì nó phải có một nhân tử bậc 1 và do đó $f$ phải có nghiệm hữu tỉ. Nói riêng vì hệ số bậc cao nhất bằng 1 nên nghiệm hữu tỉ này phải là nghiệm nguyên. Dễ thấy điều này là không xảy ra. bài toán được chứng minh xong.

Định lí 5.3 (Dạng mở rộng thứ hai của tiêu chuẩn Eisenstein):

Cho $f(x)=a_nx^n+a_{n-1}x^{n-1}+…+a_1x+a_0 \in \mathbb{Z}[x]$. Giả sử tồn tại số nguyên tố $p$ thỏa mãn với một số tự nhiên $k \le n$ nào đó mà

  • p không là ước của $a_n$;
  • $p$ là ước của $a_0, …, a_{n-1}$;
  • $p^2$ không là ước của $a_k$.

Thế thì, hoặc $f(x)$ là bất khả qui, hoặc $f$ có một nhân tử bất khả qui bậc $ \le k.$

Tương tự như trên, chứng minh được dành cho bạn đọc.

6. Các bài toán áp dụng

Bài tập 6.1 (IMO 1993): Cho số tự nhiên $n>1$. Chứng minh đa thức $f(x)=x^n+5x^{n-1}+3$ là bất khả qui trên $\mathbb{Z}[x]$.

Giải

Áp dụng dạng mở rộng thứ nhất của tiêu chuẩn Eisenstein với $p=3, k=n-1$ ta có điều phải chứng minh.

Bài tập 6.2 (China TST 1994): Cho số tự nhiên $n \ge 3$ và hai số nguyên tố $p, q$ phân biệt. Tìm tất cả các số nguyên $a$ sao cho đa thức $P(x)=x^n+ax^{n-1}+pq$ bất khả qui trên $\mathbb{Z}.$

Giải

Nếu $p|a$ hoặc $q|a$ thì theo tiêu chuẩn Eisenstein $P(x)$ là bất khả qui. Xét trường hợp $p,q$ đều không là ước của $a$. Giả sử $P$ khả qui, áp dụng dạng mở rộng thứ nhất của tiêu chuẩn Eisenstein ta suy ra $P(x)$ phải có nhân tử bậc 1 và do đó $P$ có một nghiệm nguyên $x_0$.

Từ đó suy ra $pq=-x_0^{n-1}(x_0+a)$. Vì $n \ge 3$ nên $pq \ \vdots \ x_0^2$ nhưng vì $p \ne q$ nên $x_0=\pm 1.$

Vì $1+a+pq=0$ và $(-1)^n+a(-1)^{n-1}+pq=0$ nên $a=-1-pq$ hoặc $a=1+(-1)^npq.$

Bài tập 6.3 (Rumani TST 1998): Chứng minh rằng đa thức $P(x)=(x^2+x)^{2^n}+1$ là bất khả qui với mọi số tự nhiên $n$.

Giải

Bằng qui nạp ta chỉ ra rút gọn modulo 2 thì đa thức đã cho trở thành $(x^2+x+1)^{2^n}$. Chú ý rằng đa thức $x^2+x+1$ là bất khả qui modulo 2.

Giả sử $P(x)$ khả qui, tức tồn tại hai đa thức $f,h$ đơn khởi với $f, g \in \mathbb{Z}[x], deg \ f,g \ge 1$ sao cho $P(x)=f(x).g(x)$. Khi đó $\overline{f}=(x^2+x+1)^k, \overline{g}=(x^2+x+1)^{2^n-k}$ và $$f(x)=(x^2+x+1)^k+2f_0(x), g(x)=(x^2+x+1)^{2^n-k}+2g_0(x),$$

với $f_0, g_0 \in \mathbb{Z}[x]$.

Gọi $j$ là một căn bậc 3 khác 1 của đơn vị. Thay $j$ vào đẳng thức $P(x)=f(x).h(x)$ ta được $$P(j)=g(j).h(j) \Leftrightarrow 2=4f_0(j)g_0(j).$$

Từ đó suy ra $f_0(j).g_0(j)=\dfrac{1}{2}$.

Do $f_0(j)g_0(j)$ luôn viết được dưới dạng $aj+b; a, b \in \mathbb{Z}$ và đẳng thức này không thể xảy ra. Ta có điều phải chứng minh.

Một số bài toán tương tự như sau:

Bài 1: Với $n \ge 1$ là số tự nhiên, chứng minh các đa thức sau là bất khả qui trên $\mathbb{Z}$

a) $P(x)=(x^3+x)^{2^n}-3$

b) $P(x)=(x^2+ax)^{2^n}+1$ với $ a \in \mathbb{Z}$

Bài 2: Cho $p$ là một số nguyên tố có dạng $4k+3$. Chứng minh rằng với mọi số nguyên dương $n$ đa thức $P(x)=(x^2+1)^n+p$ bất khả qui trên $\mathbb{Z}[x]$.

Bài 3: Cho $p$ là một số nguyên tố và $a$ là một số nguyên không chia hết cho $p$. Chứng minh đa thức $P(x)=x^p-x+a$ bất khả qui trên $\mathbb{Z}[x].$

Bài tập 6.4 (Japan 99): Chứng minh rằng đa thức $f(x)=(x^2+1^2)(x^2+2^2)…(x^2+n^2)+1$ là bất khả qui trên $\mathbb{Z}$

Giải

Giả sử $n \ge 2$ vì trường hợp $n=1$ là tầm thường và giả sử $f(x)=g(x).h(x)$ với $ f, g \in \mathbb{Z}[x]$ và $1 \le deg \ f, g \le n-1$. Khi đó $$1=f(\pm ki)=g(\pm ki) h(\pm ki).$$

Vì $f, g \in \mathbb{Z}[x]$ nên $g(\pm ki), h(\pm ki)$ có dạng $a + bi$ . Từ đó suy ra $$1=g(ki)h(ki)=1.1=(-1).(-1)=i.(-i)=(-i).i$$

Như vậy trong tất cả $4$ trường hợp ta đều có $g(ki)=\overline{h(ki)}=h(-ki),$ với $k=1,2,…,n$. Như vậy đa thức $P(x)=g(x)-h(-x)$ có $2n$ nghiệm phân biệt nhưng có bậc nhỏ hơn $2n$ nên là đa thức 0 và do đó $g(x)=h(-x)$. Suy ra $\deg(g)=\deg(h)=n $.

Vì $f$ đơn khởi nên $g,h$ cũng đơn khởi. Khi đó $g^2-h^2$ có bậc không quá $2n-1$ nhưng lại có ít nhất $2n$ nghiệm $ki$, với $i \in \{-n, -n-1,…,-1,1,…,n\}$ nên $g^2=h^2$.

Nhưng dễ thấy $g=-h$ không xảy ra do đó $g\equiv h$. Khi đó $$f(x)=g(x)^2 \Rightarrow f(0)=(g(0))^2=(n!)^2+1,$$ vô lí. Bài toán được chứng minh xong.

Ta có bài toán tổng quát hơn là: Cho $p$ là một số nguyên tố. Chứng minh rằng với mỗi số tự nhiên $n$ đa thức $$P(x)=(x^p+1^2)(x^p+2^2)…(x^p+n^2)+1$$ bất khả qui trên $\mathbb{Z}[x].$

Bài tập 6.5: Cho $m, n, a$ là các số nguyên dương và số nguyên tố $p$ thỏa mãn $p<a-1$. Chứng minh rằng đa thức $P(x)=x^m(x-a)^n+p$ bất khả qui trên $\mathbb{Z}$.

Giải

Giả sử $P(x)$ khả qui và $P(x)=G(x).H(x)$, với $G, H \in \mathbb{Z}[x]$. Vì $P(0)=p$ nên $|G(0)|=1$ hoặc $|H(0)|=1$. Không mất tổng quát ta giả sử $G(x)=x^k+a_{k-1}x^{k-1}+…+a_0$ và $|G(0)|=1, m+n-1 \ge k \ge 1.

Gọi $x_1,…,x_k$ là nghiệm của $G(x)$. Ta viết $G(x)$ dưới dạng $G(x)=(x-x_1)…(x-x_k)$, dẫn đến $|x_1…x_k|=1$ và $P(x_i)=0 \Leftrightarrow x_i^m(x_i-a)^n=-p$.

Cho $i=\overline{1,k}$ và nhân các vế của đẳng thức lại ta được $$ |G(x)|^n=p^k \ \text{nên} \ |G(a)|^n=p^k.$$

Mặt khác $P(a)=G(a).H(a)=p$ nên ta suy ra $|G(a)|=p$. Do đó $|G(a)-G(0)|=p \pm 1$ chia hết cho $a$.

Vì thế nên $p-1 \ge a$ hoặc $p+1 \ge a$ (mâu thuẫn với giả thiết $p<a-1$).

Vậy $P(x)$ là bất khả qui.

Bài tập 6.6 (Rumani 1999): Cho số nguyên $a$ và số nguyên dương $n$ và $p$ là một số nguyên tố thoả $p>|a|+1$. Chứng minh rằng đa thức $P(x)=x^n+ax+p$ bất khả qui trên $\mathbb{Z}[x]$.

Giải

Giả sử $P(x)=g(x).h(x)$, với $g, h \in \mathbb{Z}[x]$ và $1 \le deg f, deg g \le n-1$. Vì $P(0)=p=g(0).h(0)$ nên không mất tổng quát giả sử $g(0)=\pm 1, h(0)= \pm p.$

Khi đó $g(x)=\pm x^m+T(x)\pm 1, T \in \mathbb{Z}[x], deg T \le m-1.$

Gọi $z_1, z_2,\ldots,z_m$ là nghiệm của $g(x)$. Theo định lí Viet $1=|g(0)|=|z_1z_2…z_m|$ nên $|z_i| \le 1, i=1,2,…,m.$

Lại có $0=f(z_i)=z_i^n+az_i+p$ nên $$p=-z_i^n+az_i \le |z_i|^n+|a|.|z_i| \le 1+|a|,$$

vô lí. Vậy ta có điều phải chứng minh.

Bài tập 6.7: Cho $p, q$ là hai số nguyên tố lẻ sao cho $q$ không là ước của $p-1$ và gọi $a_1, a_2,…,a_n$ là các số nguyên phân biệt sao cho $q|(a_i-a_j)$ với mọi cặp $(i,j)$. Chứng minh rằng $$P(x)=(x-a_1)(x-a_2)…(x-a_n)-p$$ là bất khả qui trên $\mathbb{Z}[x]$ với mọi $n \ge 2.$

Giải

Giả sử $f$ khả qui trên $\mathbb{Z}[x]$. Khi đó tồn tại $Q(x), R(x) \in \mathbb{Z}[x]$ sao cho $Q(x)R(x)=P(x)$ và $1 \le deg Q(x), \le deg R(x).$ Nói riêng $degQ(x) \le \dfrac{n}{2}$.

Ta có $Q(a_i)=R(a_i)=-p,$ với $1 \le i \le n$ từ đó suy ra $Q(a_i), R(a_i) \in \{-1,1,-p,p \}$ với mọi $1 \le i \le n$. Với mọi hằng số $c$ đa thức $Q(x)-c$ nhận nhiều nhất $degQ(x) \le \dfrac{n}{2}$ nghiệm. Do đó $Q(a_i)$ nhận ít nhất hai giá trị phân biệt (và ít nhất $3$ giá trị phân biệt nếu $degQ(x)< \dfrac{n}{2}$).

Vì $q|(a_i-a_j)$ nên $q|(Q(a_i)-Q(a_j)$. Để ý rằng $q$ là số nguyên tố lẻ nên $Q(a_i)$ không thể nhận hai giá trị $1$ và $-1$ (vì nếu ngược lại thì $q|1-(-1)=2$). Tương tự $Q$ cũng không thể nhận hai giá trị là $p -p$ vì khi đó $R(a_i)$ nhận hai giá trị là $1,-1$.

Từ giả thiết $q$ không là ước của $p-1$ suy ra $Q(a_i)$ không thể nhận hai giá trị $1$ và $p$ hoặc $-1$ và $-p$. Do đó $Q(a_i)$ chỉ có thể nhận được nhiều nhất hai giá trị là $1$ và $-p$ hoặc $-1$ và $p$. Giả sử trường hợp đầu tiên xảy ra.

Vì $Q(a_i)$ chỉ nhận hai giá trị nên $degQ(a_i)=\dfrac{n}{2}$ và $Q$ nhận mỗi giá trị $1$ và $-p$ đúng $\dfrac{n}{2}$ lần. Chia tập $(a_i)_{i=1}^n$ thành hai tập $(b_i)_{i=1}^n$ và $(c_i)_{i=1}^n$ sao cho $Q(b_i)=1$ và $Q(c_i)=-p$. Khi đó $$Q(x)=(x-b_1)(x-b_2)…(x-b_{\frac{n}{2}})+1=(x-c_1)(x-c_2)…(x-c_{\frac{n}{2}})-p.$$

Vì ta cũng có $degR(x)=\dfrac{n}{2}$ nên $R(a_i)=-p$ khi $Q(a_i)=1$. Do đó $$R(x)=(x-b_1)(x-b_2)…(x-b_{\frac{n}{2}})-p=(x-c_1)(x-c_2)…(x-c_{\frac{n}{2}})+1.$$

Nhưng khi đó để ý rằng trong đẳng thức thứ nhất cho ta $Q(x)-R(x)=1+p$ còn đẳng thức thứ 2 cho ta $Q(x)-R(x)=-p-1$ điều đó dẫn đến $p=-1$. Vô lí. Bài toán được chứng minh xong.

Bài tập 6.8: Tìm tất cả các cặp số nguyên dương $(m,n)$ sao cho đa thức $$P(x,y)=(x+y)^2(mxy+n)+1$$ khả qui trên $\mathbb{Z}[x,y]$. Khi đó hãy phân tích $f$ thành các nhân tử bất khả qui.

Giải

Đặt $S=x+y$ ta viết lại $f$ dưới dạng $$f(x,S)=S^2(mx(S-x)+n)+1=-mS^2x^2+mS^3x+(nS^2+1).$$

Ta xem $f$ là một tam thức bậc 2 theo biến $x$. Khi đó $f$ khả qui khi và chỉ khi $f$ phân tích được thành tích của hai đa thức bậc nhất. Khi đó biệt thức $$\Delta =m^2S^6+4mS^2(nS^2+1)=mS^2(mS^4+4nS^2+4)$$ là một bình phương.

Dễ thấy điều này xảy ra khi và chỉ khi $m=n^2$, lúc này $$\Delta= (nS(nS^2+2))^2$$ và $f$ có hai nghiệm là $x=\dfrac{-1}{nS}$ hoặc $x=\dfrac{nS^2+1}{nS}$.

Khi đó $$f(x)=(nSx+1)(-nSx+nS^2+1)=(nx^2+nxy+1)(ny^2+nxy+1).$$

7. Bài tập tự luyện

Bài 1: Với $n \ge 2$ là một số nguyên và $r=\sqrt[n]{2}$. Chứng minh rằng không tồn tại các số hữu tỷ $a_0, a_1,…,a_{n-1}$ không đồng thời bằng $0$ sao cho $$ a_0+a_1r+a_2r^2+…+a_{n-1}r^{n-1}=0 $$

Bài 2: Tìm số nguyên dương $n$ nhỏ nhất sao cho đa thức $P(x)=x^{n-4}+4n$ có thể phân tích được thành tích của 4 đa thức hệ số nguyên và không là đa thức hằng.

Bài 3: Cho $P(x), Q(x)$ là hai đa thức đơn khởi, bất khả quy trên trường số hữu tỷ. Giả sử $P, Q$ có hai nghiệm tương ứng là $\alpha, \beta$ sao cho $\alpha +\beta$ là số hữu tỷ. Chứng minh $P^2(x)-Q^2(x)$ có nghiệm hữu tỷ.

Bài 4: Chứng minh đa thức $P(x)=(1+x+x^2+…+x^n)^2-x^n$ khả qui trên $\mathbb{Z}[x].$

Bài 5: Chứng minh rằng đa thức $P(x)=x^n+4$ khả qui trên $\mathbb{Z}$ khi và chỉ khi $n$ là bội của $4.$

Bài 6 (IMO Longlist 1989): Cho $n \ge 4$ và các số nguyên phân biệt $a_1,a_2,…,a_n$. Chứng minh đa thức $$P(x)=(x-a_1)(x-a_2)…(x-a_n)-2$$ bất khả qui trên $\mathbb{Q}[x].$

Bài 7 (VMO 2014): Cho $n$ là số nguyên dương. Chứng minh rằng đa thức $P(x)=(x^2-7x+6)^n+13$ không thể biểu diễn được thành tích của $n+1$ đa thức khác hằng với hệ số nguyên.

Bài 8: Chứng minh rằng đa thức $x^n-x-1$ bất khả qui trên $\mathbb{Q}[x]$, với mọi $n \ge 2.$

Bài 9: Cho $n>m>1$ là hai số nguyên lẻ. Chứng minh đa thức $P(x)=x^n+x^m+x+1$ bất khả qui trên $\mathbb{Z}[x]$.

Bài 10: Cho $p$ là số nguyên tố. Chứng minh rằng đa thức $$P(x)=x^{p-1}+2x^{p-2}+…+(p-1)x+p$$ bất khả qui trên $\mathbb{Z}$.

Bài 11: Cho đa thức $P(x)=a_nx^n+a_{n-1}x^{n-1}+…+a_1x+a_0 \in \mathbb{Z}[x], (a_n \ne 0, n \ge 2)$. Chứng minh rằng tồn tại vô số số nguyên tố $k$ sao cho đa thức $P(x)+k$ bất khả qui.

Bài 12: Tìm tất cả các số nguyên $n$ sao cho đa thức $P(x)=x^5-nx-n-2$ là khả qui trên $\mathbb{Z}[x]$.

Bài 13: Cho $p$ là một số nguyên tố và $n$ là một số nguyên nhỏ hơn 4. Chứng minh rằng nếu $a$ là một số nguyên không chia hết cho $p$ thì đa thức $P(x)=ax^n-px^2+px+p^2$ bất khả qui trên $\mathbb{Z}[x].$

Bài 14: Cho $p$ là số nguyên tố. Chứng minh rằng đa thức $P(x)=x^p+(p-1)!$ bất khả qui trên $\mathbb{Z}[x]$.

Bài 15: Tồn tại hay không đa thức $f \in \mathbb{Q}[x]$ sao cho $f(1) \ne -1$ và $x^nf(x)+1$ là khả qui với mọi $n \in \mathbb{N}$.

Bài 16: Cho $a$ là một số nguyên dương và $p \ge 2 $ là một số nguyên tố thỏa mãn $(a,p)=1$. Chứng minh rằng đa thức $P(x)=x^p-mx+a$ bất khả qui trên $\mathbb{Z}[x]$ với $m \equiv \ 1 \ (mod \ p)$.

Bài 17: Cho $p$ là một số nguyên tố lẻ. Chứng minh đa thức $P(x)= \sum \limits_{i=0}^{p-2}(p-1-i)x^i$ bất khả qui trên $\mathbb{Q}[x].$

Bài 18 (Rumani TST 2003): Cho $P(x) \in \mathbb{Z}[x]$ là một đa thức monic bất khả qui trên $\mathbb{Z}[x]$ sao cho $P(0)$ không là số chính phương. Chứng minh rằng $Q(x)=P(x^2)$ cũng bất khả qui trên $\mathbb{Z}[x].$

Bài 19 (China TST 2006): Cho số nguyên $n \ge 2$. Chứng minh rằng tồn tại đa thức $P(x)=x^n+a_{n-1}x^{n-1}+…+a_1x+a_0$ thỏa mãn

a) $a_0, a_1,…,a_{n-1}$ khác 0.

b) $P(x)$ bất khả qui.

c) Với mọi số nguyên $x$ thì $|P(x)|$ không là số nguyên tố.

Bài 20: Biết $f \in \mathbb{Z}[x]$ là một đa thức bất khả qui có bậc lẻ và lớn hơn 3. Giả sử rằng các nghiệm của $P$ đều có modun lớn hơn 1 và $f(0)$ không có ước chính phương. Chứng minh rằng đa thức $g(x)=f(x^3)$ cũng là đa thức bất khả qui.

Bài 21: Cho $f \in \mathbb{Z}[x]$ là một đa thức monic với bậc lớn hơn 1. Giả sử $f(x^n)$ bất khả qui trên $\mathbb{Z}[x]$ với mọi $n \ge 2$. Hỏi $f$ có bất khả qui trên $\mathbb{Z}[x]$ hay không?

Bài 22: Cho $1 \ne f \in \mathbb{Z}[x]$ sao cho có vô hạn số nguyên $a$ thỏa $f(x^2+ax)$ bất khả qui trên $\mathbb{Q}[x]$. Hỏi $f$ có bất khả qui trên $\mathbb{Q}[x]$ hay không?

Bài 23: Cho $f(x) \ne \pm x$ là một đa thức bất khả qui trên $\mathbb{Z}[x]$. Hỏi $f(x.y)$ có bất khả qui trên $\mathbb{Z}[x,y]$ hay không?

Tài liệu tham khảo

[1] Nguyễn Tiến Quang, NXB Giáo dục, Đại số đại cương

 

[2] Đoàn Duy Cường, 2015, Bài giảng bồi dưỡng giáo viên chuyên toán năm

 

[3] Nguyễn Chu Gia Vượng,2015,  Đa thức bất khả qui

 

[4] Exploration-Creativity 2012,  Irreducible polynomials

 

[5] Yufei Zhao, Integer polynomial 

 

[6] Dusan Djukic, Polynomials in one variable 

 

[7] Gabriel D.Carroll, Polynomials 

 

[8] Victor V.Prasolov, Polynomials

 

[9] Titu Andresscu, Gabriel Dospinescu, Problems from the book

 

[10] U298, Mathematical Reflections

 

[11] https://artofproblem.../community/c89 




#737436 Đề thi học sinh giỏi quốc gia (VMO) năm 2023

Gửi bởi Saturina trong 25-02-2023 - 23:04

Một sol mà mình kiếm được:

Đề và đáp án môn Toán thi học sinh giỏi quốc gia năm 2023 - VnExpress

https://vnexpress.ne...ai-4574770.html




#737326 Tìm tất cả các giá trị của $a,b$ sao cho phương trình $x^...

Gửi bởi Saturina trong 18-02-2023 - 22:32

Tìm tất cả các giá trị của $a,b$ sao cho phương trình $x^{3}+ax^{2}+bx+3a=0$ có các nghiệm đều là số nguyên dương




#735935 $\sum_{cyc}{\dfrac{a}{b}...

Gửi bởi Saturina trong 28-11-2022 - 20:34

Phương pháp S-S, có

$\sum_{cyc}^{}\frac{a}{b} -3 = \frac{(a-b)^{2}}{ab} + \frac{(a-c)(b-c)}{ac}$
$\frac{ab+bc+ca}{a^{2}+b^{2}+c^{2}} -1=-\frac{(a-b)^{2}+(a-c)b-c)}{a^{2}+b^{2}+c^{2}}$

Do đó BĐT bài cho tương đương:
$M(a-b)^{2} + N(a-c)(b-c)\geq 0$ với M, N lần lượt là: $\frac{1}{ab}-\frac{3}{2(a^{2}+b^{2}+c^{2})}$ và $\frac{1}{ac}-\frac{3}{2(a^{2}+b^{2}+c^{2})}$
Mà dễ dàng cm dc $M\geq 0,N\geq 0$
Ta có đpcm, dấu bằng xảy ra khi a=b=c

Phương pháp S-S là phương pháp gì thế ạ?




#735931 $\sum_{cyc}{\dfrac{a}{b}...

Gửi bởi Saturina trong 28-11-2022 - 12:44

Với mọi $a,b,c > 0$, chứng minh rằng:

$$\dfrac{a}{b}+\dfrac{b}{c}+\dfrac{c}{a}+\dfrac{3}{2}.\dfrac{ab+bc+ca}{a^2+b^2+c^2} \ge \dfrac{9}{2}$$